🎧 New: AI-Generated Podcasts Turn your study notes into engaging audio conversations. Learn more

HESI OB Nursing Concepts (Arizona State University) PDF

Loading...
Loading...
Loading...
Loading...
Loading...
Loading...
Loading...

Summary

This document contains practice questions for obstetrics nursing from Arizona State University (no exam board or year listed). The questions cover various topics in the field, including the stages of labor, fetal monitoring, and pain management.

Full Transcript

lOMoARcPSD|15450111 Ob-last1 - hesi ob Nursing Concepts (Arizona State University) Scan to open on Studocu Studocu is not sponsored or endorsed by any college or university Downloaded by Craig Cman ([email protected]) lOMoARcPSD|15450111 A client is admitted to the labor and delivery unit with co...

lOMoARcPSD|15450111 Ob-last1 - hesi ob Nursing Concepts (Arizona State University) Scan to open on Studocu Studocu is not sponsored or endorsed by any college or university Downloaded by Craig Cman ([email protected]) lOMoARcPSD|15450111 A client is admitted to the labor and delivery unit with contractions that are 3-5 minutes apart, lasting 60-70 seconds. She reports that she is leaking fluid. A vaginal exam reveals that her cervix is 80 percent effaced and 4 cm dilated and a -1 station. The LPN/LVN knows that the client is in which phase and stage of labor? A) Latent phase, First Stage B) Active Phase of First Stage C) Latent phase of Second Stage D) Transition B) Active Phase of First Stage To assess uterine contractions the LPN/LVN would A) Asses duration from the beginning of the contraction to the peak of the same contraction, frequency by measuring the time between the beginning of one contraction to the beginning of the next contraction. B) Assess frequency as the time between the end of one contraction and the beginning of the next contraction, duration as the length of time from the beginning to the end of contractions, and palpate the uterus for strength C) Assess duration from beginning to end of each contraction. Assess the strength of the contraction by the external fetal monitor reading.Measure frequency by measuring the beginning of one contraction to another. D) Assess duration from beginning to end of each contraction., frequency by measuring the time between the beginnings of contractions, and palpate the fundus of the uterus for strength. D) Assess duration from beginning to end of each contraction., frequency by measuring the time between the beginnings of contractions, and palpate the fundus of the uterus for strength. Which basic type of pelvis includes the correct description and percentage of occurrence in women? Downloaded by Craig Cman ([email protected]) lOMoARcPSD|15450111 A) Platypelloid: flattened, wide, shallow; 3% B) Anthropoid: resembling the ape; narrower; 10% C) Android: resembling the male; wider oval; 15% D) Gynecoid: classic female; heart shaped; 75% A) Platypelloid: flattened, wide, shallow; 3% AD What position would be least effective when gravity is desired to assist in fetal descent? A) Lithotomy B) Walking C) Kneeling D) Sitting A) Lithotomy The factors that affect the process of labor and birth, known commonly as the five Ps, include all EXCEPT: A) Passageway. B) Powers. C) Passenger. D) Pressure. D) Pressure. While evaluating an external monitor tracing of a woman in active labor, the nurse notes that the fetal heart rate (FHR) for five sequential contractions begins to decelerate late in the contraction, with the nadir of the decelerations occurring after the peak of the contraction. The LPN/LVN first priority is to: A) Notify the care provider. B) Assist with amnioinfusion Downloaded by Craig Cman ([email protected]) lOMoARcPSD|15450111 C) Change the woman's position D) Insert a scalp electrode C) Change the woman's position During labor a fetus with an average heart rate of 175 beats/min over a 15minute period would be considered to have: A) A normal baseline heart rate. B) Bradycardia. C) Hypoxia. D) Tachycardia. D) Tachycardia. As a perinatal LPN/LVN you realize that a fetal heart rate that is tachycardic, is bradycardic, or has late decelerations with loss of variability is nonreassuring and is associated with A) Cord compression B) Hypotension C) Hypoxemia/acidemia D) Maternal drug use. C) Hypoxemia/acidemia 9) The LPN/LVN providing care for the laboring woman should understand that amnioinfusion is used to treat: A) Fetal tachycardia. B) Fetal bradycardia. C) Variable decelerations D) Late decelerations. C) Variable decelerations he most common cause of decreased variability in the fetal heart rate (FHR) that lasts 30 minutes or less is: A) Fetal hypoxemia Downloaded by Craig Cman ([email protected]) lOMoARcPSD|15450111 B) Fetal sleep cycles C) Altered cerebral blood flow. D) Umbilical cord compression. B) Fetal sleep cycles While evaluating an external monitor tracing of a woman in active labor whose labor is being induced, the nurse notes that the fetal heart rate (FHR) begins to decelerate in a slow curve at the onset of several contractions and returns to baseline before each contraction ends. The LPN/LVN should: A) Insert an internal monitor B) Document the finding in the client's record. C) Discontinue the oxytocin infusion D) Change the woman's position B) Document the finding in the client's record. What three measures should the nurse implement to provide intrauterine resuscitation? Select the response that best indicates the priority of actions that should be taken. A) Reposition the mother, increase intravenous (IV) fluid, and provide oxygen via face mask. B) Perform a vaginal examination, reposition the mother, and provide oxygen via face mask. C) Administer oxygen to the mother, increase IV fluid, and notify the care provider. D) Call the provider, reposition the mother, and perform a vaginal examination A) Reposition the mother, increase intravenous (IV) fluid, and provide oxygen via face mask. AD When using intermittent auscultation (IA) to assess uterine activity, LPN/LVN should be aware that: Downloaded by Craig Cman ([email protected]) lOMoARcPSD|15450111 A) The resting tone between contractions is described as either placid or turbulent B) The examiner's hand should be placed over the fundus before, during, and after contractions. C) The frequency and duration of contractions is measured in seconds for consistency D) Contraction intensity is given a judgment number of 1 to 7 by the nurse and client together. B) The examiner's hand should be placed over the fundus before, during, and after contractions. Perinatal LPN/LVN are legally responsible for: A) Applying the external fetal monitor and notifying the care provider. B) Correctly interpreting fetal heart rate (FHR) patterns, initiating appropriate nursing interventions, and documenting the outcomes. C) Greeting the client on arrival, assessing her, and starting an intravenous line. D) Making sure that the woman is comfortable B) Correctly interpreting fetal heart rate (FHR) patterns, initiating appropriate nursing interventions, and documenting the outcomes. The LPN/LVN providing care for the laboring woman should understand that late fetal heart rate (FHR) decelerations are caused by: A) Altered cerebral blood flow B) Spontaneous rupture of membranes C) Uteroplacental insufficiency D) Umbilical cord compression C) Uteroplacental insufficiency Downloaded by Craig Cman ([email protected]) lOMoARcPSD|15450111 The LPN/LVN providing care for the laboring woman should understand that variable fetal heart rate (FHR) decelerations are caused by: A) Umbilical cord compression. B) Altered fetal cerebral blood flow C) Fetal hypoxemia. D) Uteroplacental insufficiency A) Umbilical cord compression. Which of the following is NOT a reassuring component of the fetal heart rate A) FHR of 114 B) Accelerations of the FHR C) Moderate Variability D) Absent FHR Variability D) Absent FHR Variability You are evaluating the fetal monitor tracing of your client, who is in active labor. Suddenly you see the fetal heart rate (FHR) drop from its baseline of 125 down to 80. You reposition the mother, provide oxygen, increase intravenous (IV) fluid, and perform a vaginal examination. The cervix has not changed. Five minutes have passed, and the fetal heart rate remains in the 80s. What additional nursing measures should you take? A) Call for help and Notify the care provider immediately B) Start pitocin C) Have her empty her bladder D) Insert a Foley catheter A) Call for help and Notify the care provider immediately What is an advantage of external electronic fetal monitoring? A) Once correctly applied by the nurse, the transducer need not be repositioned even when the woman changes positions. Downloaded by Craig Cman ([email protected]) lOMoARcPSD|15450111 B) The tocotransducer can measure and record the frequency, regularity, intensity, and approximate duration of uterine contractions (UCs). C) The external EFM does not require rupture of membranes or introduction of scalp electrode or IUPC which may introduce risk of infection or fetal scarring. D) The external EFM can accurately record FHR all the time. C) The external EFM does not require rupture of membranes or introduction of scalp electrode or IUPC which may introduce risk of infection or fetal scarring. A number of methods to assist in the assessment of fetal well-being have been developed for use in conjunction with electronic fetal monitoring. These various technologies assist in supporting interventions for a nonreassuring fetal heart rate pattern when necessary. The labor and delivery nurse should be aware that one of these modalities, fetal oxygen saturation monitoring, includes the use of: A) Fetal blood sampling B) Umbilical cord acid-base determination C) Fetal pulse oximetry. D) A fetal acoustic stimulator. C) Fetal pulse oximetry. The LPN/LVN caring for the woman in labor should understand that maternal hypotension can result in: A) Uteroplacental insufficiency. B) Spontaneous rupture of membranes C) Fetal dysrhythmias. D) Early decelerations. A) Uteroplacental insufficiency. AD Downloaded by Craig Cman ([email protected]) lOMoARcPSD|15450111 The LPN/LVN providing care for the laboring woman should understand that accelerations with fetal movement: A) Are caused by umbilical cord compression B) Are caused by uteroplacental insufficiency C) Warrant close observation D) Are reassuring. D) Are reassuring. A woman in active labor receives an analgesic, an opioid agonist. Which medication relieves severe, persistent, or recurrent pain; creates a sense of well-being; overcomes inhibitory factors; and may even relax the cervix but should be used cautiously in women with cardiac disease? A) Meperidine (Demerol) B) Promethazine (Phenergan) C) Butorphanol tartrate (Stadol) D) Nalbuphine (Nubain) A) Meperidine (Demerol) A laboring woman received meperidine (Demerol) intravenously 90 minutes before she gave birth. Which medication should be available to reduce the postnatal effects of Demerol on the neonate? A) Fentanyl (Sublimaze) B) Promethazine (Phenergan) C) Naloxone (Narcan) D) Nalbuphine (Nubain) C) Naloxone (Narcan) A woman in labor has just received an epidural block. The most important nursing intervention is to: Downloaded by Craig Cman ([email protected]) lOMoARcPSD|15450111 A) Limit parenteral fluids. B) Monitor the fetus for possible tachycardia C) Monitor the maternal blood pressure for possible hypotension. D) Monitor the maternal pulse for possible bradycardia C) Monitor the maternal blood pressure for possible hypotension. A woman is experiencing back labor and complains of intense pain in her lower back. An effective relief measure would be to use: A) Counter pressure against the sacrum B) Pant-blow (breaths and puffs) breathing techniques C) Effleurage. D) Conscious relaxation or guided imagery. A) Counter pressure against the sacrum A woman has requested an epidural for her pain. She is 5 cm dilated and 100% effaced. The baby is in a vertex position and is engaged. The LPN/LVN increases the woman's intravenous fluid for a pre-procedural bolus. She reviews her laboratory values and notes that the woman's hemoglobin is 12 g/dl, hematocrit is 38%, platelets are 67,000, and white blood cells (WBCs) are 12,000/mm3. Which factor would contraindicate an epidural for the woman? A) She is too far dilated B) She is anemic. C) She has thrombocytopenia D) She is septic C) She has thrombocytopenia The role of the LPN/LVN with regard to informed consent is to: A) Inform the client about the procedure and have her sign the consent form. B) Act as a client advocate and help clarify the procedure and the options. Downloaded by Craig Cman ([email protected]) lOMoARcPSD|15450111 C) Call the physician to see the client D) Witness the signing of the consent form. B) Act as a client advocate and help clarify the procedure and the options. With regard to systemic analgesics administered during labor, LPN/LVN should be aware that: A) Systemic analgesics cross the maternal blood-brain barrier as easily as they do the fetal blood-brain barrier. B) Effects on the fetus and newborn can include decreased alertness and delayed sucking. C) Intramuscular administration (IM) is preferred over intravenous (IV) administration. D) IV patient-controlled analgesia (PCA) results in increased use of an analgesic. B) Effects on the fetus and newborn can include decreased alertness and delayed sucking. With regard to spinal and epidural (block) anesthesia, LPN/LVN should know that: A) This type of anesthesia is commonly used for cesarean births but isnot suitable for vaginal births B) The incidence of after-birth headache is higher with spinal blocks than epidurals. C) Epidural blocks allow the woman to move freely D) Spinal and epidural blocks are never used together. B) The incidence of after-birth headache is higher with spinal blocks than epidurals. Maternal hypotension is a potential side effect of regional anesthesia and analgesia. What nursing interventions could you use to raise the client's blood pressure? Choose all that apply. Downloaded by Craig Cman ([email protected]) lOMoARcPSD|15450111 A) Place the woman in a supine position. B) Place the woman in a lateral position. C) Increase intravenous (IV) fluids. D) Continuous Fetal Monitor E) Administer ephedrine per MD order B) Place the woman in a lateral position. C) Increase intravenous (IV) fluids. E) Administer ephedrine per MD order Nursing care measures are commonly offered to women in labor. Which nursing measure reflects application of the gate-control theory? A) Massaging the woman's back B) Changing the woman's position C) Giving the prescribed medication D) Encouraging the woman to rest between contractions A) Massaging the woman's back A woman in the active phase of the first stage of labor is using a shallow pattern of breathing, which is about twice the normal adult breathing rate. She starts to complain about feeling lightheaded and dizzy and states that her fingers are tingling. The LPN/LVN should: A) Notify the woman's physician. B) Tell the woman to "calm down" and slow the pace of her breathing. C) Administer oxygen via a mask or nasal cannula. D) Help her breathe into a paper bag D) Help her breathe into a paper bag With regard to a pregnant woman's anxiety and pain experience, LPN/LVN should be aware that: Downloaded by Craig Cman ([email protected]) lOMoARcPSD|15450111 A) Even mild anxiety must be treated. B) Severe anxiety increases tension, which increases pain, which in turn increases fear and anxiety, and so on. C) Anxiety may increase the perception of pain, but it does not affect the mechanism of labor. D) Women who have had a painful labor will have learned from the experience and have less anxiety the second time because of increased familiarity. B) Severe anxiety increases tension, which increases pain, which in turn increases fear and anxiety, and so on. Maternity nurse often have to answer questions about the many, sometimes unusual ways people have tried to make the birthing experience more comfortable. For instance, LPN/LVN should be aware that: A) Music supplied by the support person has to be discouraged because it could disturb others or upset the hospital routine. B) Women in labor can benefit from sitting in a bathtub, but they must limit immersion to no longer than 15 minutes at a time. C) Effleurage is permissible, but counter pressure is almost always counterproductive. D) Electrodes attached to either side of the spine to provide mild- intensity electrical impulses facilitate the release of endorphins. D) Electrodes attached to either side of the spine to provide mild- intensity electrical impulses facilitate the release of endorphins. Your patient is a nulliparous woman, requesting pain relief. You examine her and she is 8 cm. What is the best option for pain relief at this point? A) Demerol B) Spinal Downloaded by Craig Cman ([email protected]) lOMoARcPSD|15450111 C) Epidural D) Stadol C) Epidural A primiparous woman is in the triage room being evaluated for labor. She has been having contractions for 2 days, has slept little and is feeling exhausted. On cervical exam she is 1.5 cm dilated, 50% effaced, -1 station - which is not changed from a day ago. Contractions are irregular, 30-40 secs long. Which of the following is the best option for her? A) Offer morphine IM, and a sedative to help her sleep. B) Admit her and give her an epidural. C) Tell her to go home, relax D) Give her a couple of seconal to help her sleep. A) Offer morphine IM, and a sedative to help her sleep. Which of the following is NOT a reason to come to labor and birth. A) The patient is 39 weeks with second baby. She has been having contractions for 2 hours. Contractions are getting longer and stronger and closer together. B) The patients says she has noticed greenish fluid leaking from her vagina. She is 41.5 weeks pregnant and not having contractions. C) The patient is 40 weeks and has contractions that are 8-10 minutes apart, 30 seconds long and been that way for 8 hours. D) The patient has not felt the baby move for 8 hours, despite drinking cold fluids, and nudging the baby with her hand. C) The patient is 40 weeks and has contractions that are 8-10 minutes apart, 30 seconds long and been that way for 8 hours. What is an expected characteristic of amniotic fluid? A) Deep yellow color Downloaded by Craig Cman ([email protected]) lOMoARcPSD|15450111 B) Pale, straw color with small white particles C) Acidic result on a Nitrazine test D) Absence of ferning B) Pale, straw color with small white particles AD The uterine contractions of a woman early in the active phase of labor are assessed by an internal uterine pressure catheter (IUPC). The nurse notes that the intrauterine pressure at the peak of the contraction ranges from 65 to 70 mm Hg and the resting tone range is 6 to 10 mm Hg. The uterine contractions occur every 3 to 4 minutes and last an average of 55 to 60 seconds. On the basis of this information, the LPN should: A) Notify the woman's primary health care provider immediately B) Prepare to administer an oxytocic to stimulate uterine activity C) Document the findings because they reflect the expected contraction pattern for the active phase of labor. D) Prepare the woman for the onset of the second stage of labor. C) Document the findings because they reflect the expected contraction pattern for the active phase of labor. A multiparous woman has been in labor for 8 hours. Her membranes have just ruptured. The nurse initial response would be to: A) Prepare the woman for imminent birth B) Notify the woman's primary health care provider. C) Document the characteristics of the fluid. D) Assess the fetal heart rate and pattern D) Assess the fetal heart rate and pattern Through vaginal examination the nurse determines that a woman is 4 cm dilated, and the external fetal monitor shows uterine contractions every 3.5 to 4 minutes. The LPN/LVN would report this as: Downloaded by Craig Cman ([email protected]) lOMoARcPSD|15450111 A) First stage, latent phase B) First stage, active phase C) First stage, transition phase D) Second stage, latent phase B) First stage, active phase The LPN/LVN expects to administer an oxytocic (e.g., Pitocin, Methergine) to a woman after expulsion of her placenta to: A) Relieve pain. B) Stimulate uterine contraction C) Prevent infection D) Facilitate rest and relaxation. B) Stimulate uterine contraction Vaginal examinations should be performed by the LPN/LVN under all of these circumstances EXCEPT: A) An admission to the hospital at the start of labor. B) When accelerations of the fetal heart rate (FHR) are noted. C) On maternal perception of perineal pressure or the urge to bear down. D) When membranes rupture B) When accelerations of the fetal heart rate (FHR) are noted. With regard to a woman's intake and output during labor, LPN/LVN should be aware that: A) The tradition of restricting the laboring woman to clear liquids and ice chips is being challenged because regional anesthesia is used more often than general anesthesia and studies are not showing harm from drinking fluids in labor. B) Intravenous (IV) fluids usually are necessary to ensure that the laboring woman stays hydrated. Downloaded by Craig Cman ([email protected]) lOMoARcPSD|15450111 C) Routine use of an enema empties the rectum and is very helpful for producing a clean, clear delivery. D) When a nulliparous woman experiences the urge to defecate, it often means birth will follow quickly A) The tradition of restricting the laboring woman to clear liquids and ice chips is being challenged because regional anesthesia is used more often than general anesthesia and studies are not showing harm from drinking fluids in labor. For women who have a history of sexual abuse, a number of traumatic memories may be triggered during labor. The woman may fight the labor process and react with pain or anger. Alternately she may become a passive player and emotionally absent herself from the process. The nurse is in a unique position of being able to assist the client to associate the sensations of labor with the process of childbirth and not the past abuse. The nurse can implement a number of care measures to help her client view the childbirth experience in a positive manner. Which intervention would be key for the LPN/LVN to use while providing care? A) Telling the client to relax and that it won't hurt much B) Limiting the number of procedures that invade her body C) Reassuring the client that as the nurse you know what is best D) Allowing unlimited care providers to be with the client B) Limiting the number of procedures that invade her body Concerning the third stage of labor, LPN/LVN should be aware that: A) The placenta eventually detaches itself from a flaccid uterus B) An active approach to managing this stage of labor reduces the risk of excessive bleeding C) It is important that the dark, roughened maternal surface of the placenta Downloaded by Craig Cman ([email protected]) lOMoARcPSD|15450111 appear before the shiny fetal surface. D) The major risk for women during the third stage is a rapid heart rate. B) An active approach to managing this stage of labor reduces the risk of excessive bleeding When planning care for a laboring woman whose membranes have ruptured, the LPN/LVN recognizes that the woman's risk for has increased. A) Intrauterine infection B) Hemorrhage C) Precipitous labor D) Supine hypotension A) Intrauterine infection When managing the care of a woman in the second stage of labor, the LPN/LVN uses various measures to enhance the progress of fetal descent. These measures include: A) Encouraging the woman to try various upright positions, including squatting and standing. Giving positive feedback about her efforts. B) Telling the woman to start pushing as soon as her cervix is fully dilated. C) Stopping the epidural anesthetic so the woman can feel the urge to push and thereby push more effectively D) Coaching the woman to use sustained, 10- to 15-second, closed- glottis bearing-down efforts with each contraction. A) Encouraging the woman to try various upright positions, including squatting and standing. Giving positive feedback about her efforts. When assessing a multiparous woman who has just given birth to an 8-pound boy, the nurse notes that the woman's fundus is firm and has become globular in shape. A gush of dark red blood comes from her vagina. The LPN/LVN concludes that: A) The placenta has separated. Downloaded by Craig Cman ([email protected]) lOMoARcPSD|15450111 B) A cervical tear occurred during the birth C) The woman is beginning to hemorrhage. D) Clots have formed in the upper uterine segment. A) The placenta has separated. LPN can help their clients by keeping them informed about the distinctive stages of labor. What description of the phases of the first stage of labor is accurate? A) Latent: Mild, regular contractions; no dilation; bloody show; duration of 2 to 4 hours B) Active: Moderate, regular contractions; 4- to 7-cm dilation; duration of 3 to 6 hours C) Lull: No contractions; dilation stable; duration of 20 to 60 minutes D) Transition: Very strong but irregular contractions; 8- to 10-cm dilation; duration of 3-4 hours B) Active: Moderate, regular contractions; 4- to 7-cm dilation; duration of 3 to 6 hours Match the degree of tear or episiotomy to its description A. Laceration that goes through the anal sphincter and the rectal wall B. a tear through part or all of the perineal muscles C. small nick in the perineum, not involving muscle D. Laceration through part or all of anal sphincter muscle 1st degree 2nd degree 3rd degree 4th degree 1st degree = C. small nick in the perineum, not involving muscle 2nd degree = B. a tear through part or all of the perineal muscles Downloaded by Craig Cman ([email protected]) lOMoARcPSD|15450111 3rd degree = D. Laceration through part or all of anal sphincter muscle 4th degree = A. Laceration that goes through the anal sphincter and the rectal wall Which of the following is true about placenta previa? A) The bleeding from placenta previa usually occurs late in pregnancy at term. B) In evaluating the bleeding, a vaginal exam would be done to determine the cause of the bleeding. C) Symptoms of placenta previa are painful frequent contractions and bright red vaginal bleeding D) Once placenta previa is diagnosed by a 20 week ultrasound, it is very likely the placenta previa will resolve in the third trimester. D) Once placenta previa is diagnosed by a 20 week ultrasound, it is very likely the placenta previa will resolve in the third trimester. What assessment is least likely to be associated with a breech presentation? A) Fetal heart tones heard at or above the maternal umbilicus B) Meconium-stained amniotic fluid C) Postterm gestation D) Preterm labor and birth C) Postterm gestation During labor, the patient at 4 cm suddenly becomes dyspneic, cyanotic, and hypotensive. The nurse must prepare immediately for: (Select all that apply.) A) Immediate vaginal delivery B) Cesarean delivery C) CPR D) McRobert's maneuver Downloaded by Craig Cman ([email protected]) lOMoARcPSD|15450111 B) Cesarean delivery C) CPR The nurse is caring for a client who had an emergency cesarean section, with her husband in attendance the day before. The baby's Apgar was 9/9. The woman and her partner had attended childbirth education classes and had anticipated having a water birth with family present. Which of the following comments by the nurse is appropriate? 1. "Sometimes babies just don't deliver the way we expect them to." 2. "With all of your preparations, it must have been disappointing for you to have had a cesarean." 3. "I know you had to have surgery, but you are very lucky that your baby was born healthy." 4. "At least your husband was able to be with you when the baby was born." 2. "With all of your preparations, it must have been disappointing for you to have had a cesarean.". A nurse has brought a 2-hour-old baby to a mother from the nursery. The nurse is going to assist the mother with the first breastfeeding experience. Which of the following actions should the nurse perform first? 1. Compare mother's and baby's identification bracelets. 2. Help the mother into a comfortable position. 3. Teach the mother about a proper breast latch. 4. Tickle the baby's lips with the mother's nipple. Downloaded by Craig Cman ([email protected]) lOMoARcPSD|15450111 1. Compare mother's and baby's identification bracelets. The obstetrician has ordered that a post-op cesarean section client's patient controlled analgesia (PCA) be discontinued. Which of the following actions by the nurse is appropriate? 1. Discard the remaining medication in the presence of another nurse. 2. Recommend waiting until her pain level is zero to discontinue the medicine. 3. Discontinue the medication only after the analgesia is completely absorbed. 4. Return the unused portion of medication to the narcotics cabinet. 1. Discard the remaining medication in the presence of another nurse. AD 학습하지 않음 (97) 아직 이 단어들을 학습하지 않았어요! 97 개 선택 A client is receiving an epidural infusion of a narcotic for pain relief after a cesarean section. The nurse would report to the anesthesiologist if which of the following were assessed? 1. Respiratory rate 8 rpm. 2. Complaint of thirst. 3. Urinary output of 250 cc/hr. 4. Numbness of feet and ankles. 1. Respiratory rate 8 rpm. A client, 2 days postoperative from a cesarean section, complains to the nurse that she has yet to have a bowel movement since the surgery. Which of the following responses by the nurse would be appropriate at this time? 1. "That is very concerning. I will request that your physician order an enema for you." 2. "Two days is not that bad. Some patients go four days or longer without a Downloaded by Craig Cman ([email protected]) lOMoARcPSD|15450111 movement." 3. "You have been taking antibiotics through your intravenous. That is probably why you are constipated." 4. "Fluids and exercise often help to combat constipation. Take a stroll around the unit and drink lots of fluid." 4. "Fluids and exercise often help to combat constipation. Take a stroll around the unit and drink lots of fluid." A post-cesarean section, breastfeeding client, whose subjective pain level is 2/5, requests her as needed (prn) narcotic analgesics every 3 hours. She states, "I have decided to make sure that I feel as little pain from this experience as possible." Which of the following should the nurse conclude in relation to this woman's behavior? 1. The woman needs a stronger narcotic order. 2. The woman is high risk for severe constipation. 3. The woman's breast milk volume may drop while taking the medicine. 4. The woman's newborn may become addicted to the medication. 2. The woman is high risk for severe constipation.. A nurse is assessing a 1-day postpartum woman who had her baby by cesarean section. Which of the following should the nurse report to the surgeon? 1. Fundus at the umbilicus. 2. Nodular breasts. 3. Pulse rate 60 bpm. 4. Pad saturation every 30 minutes. 4. Pad saturation every 30 minutes. The nurse is assessing the midline episiotomy on a postpartum client. Which of the following findings should the nurse expect to see? 1. Moderate serosanguinous drainage. 2. Well-approximated edges. Downloaded by Craig Cman ([email protected]) lOMoARcPSD|15450111 3. Ecchymotic area distal to the episiotomy. 4. An area of redness adjacent to the incision. 2. Well-approximated edges. A client, G1P1, who had an epidural, has just delivered a daughter, Apgar 9/9, over a mediolateral episiotomy. The physician used low forceps. While recovering, the client states, "I'm a failure. I couldn't stand the pain and couldn't even push my baby out by myself!" Which of the following is the best response for the nurse to make? 1. "You'll feel better later after you have had a chance to rest and to eat." 2. "Don't say that. There are many women who would be ecstatic to have that baby." 3. "I am sure that you will have another baby. I bet that it will be a natural delivery." 4. "To have things work out differently than you had planned is disappointing." 4. "To have things work out differently than you had planned is disappointing." The nurse is developing a standard care plan for postpartum clients who have had midline episiotomies. Which of the following interventions should be included in the plan? 1. Assist with stitch removal on third postpartum day. 2. Administer analgesics every four hours per doctor orders. 3. Teach client to contract her buttocks before sitting. 4. Irrigate incision twice daily with antibiotic solution. 3. Teach client to contract her buttocks before sitting.. A client, G1P1001, 1-hour postpartum from a spontaneous vaginal delivery with local anesthesia, states that she needs to urinate. Which of the following actions by Downloaded by Craig Cman ([email protected]) lOMoARcPSD|15450111 the nurse is appropriate at this time? 1. Provide the woman with a bedpan. 2. Advise the woman that the feeling is likely related to the trauma of delivery. 3. Remind the woman that she still has a catheter in place from the delivery. 4. Assist the woman to the bathroom. 4. Assist the woman to the bathroom. A nurse is assessing the fundus of a client during the immediate postpartum period. Which of the following actions indicates that the nurse is performing the skill correctly? 1. The nurse measures the fundal height using a paper centimeter tape. 2. The nurse stabilizes the base of the uterus with his or her dependent hand. 3. The nurse palpates the fundus with the tips of his or her fingers. 4. The nurse precedes the assessment with a sterile vaginal exam. 2. The nurse stabilizes the base of the uterus with his or her dependent hand. AD 13. A 1-day postpartum woman states, "I think I have a urinary tract infection. I have to go to the bathroom all the time." Which of the following actions should the nurse take? 1. Assure the woman that frequent urination is normal after delivery. 2. Obtain an order for a urine culture. 3. Assess the urine for cloudiness. 4. Ask the woman if she is prone to urinary tract infections. 1. Assure the woman that frequent urination is normal after delivery. The nurse is assessing the laboratory report on a 2-day postpartum G1P1001. The woman had a normal postpartum assessment this morning. Which of the following results should the nurse report to the primary health care provider? 1. White blood cells—12,500 cells/mm3. Downloaded by Craig Cman ([email protected]) lOMoARcPSD|15450111 2. Red blood cells—4,500,000 cells/mm3. 3. Hematocrit—26%. 4. Hemoglobin—11 g/dL 3. Hematocrit—26%. A bottlefeeding woman, 11⁄2 weeks postpartum from a vaginal delivery, calls the obstetric office to state that she has saturated 2 pads in the past 1 hour. Which of the following responses by the nurse is appropriate? 1. "You must be doing too much. Lie down for a few hours and call back if the bleeding has not subsided." 2. "You are probably getting your period back. You will bleed like that for a day or two and then it will lighten up." 3. "It is not unusual to bleed heavily every once in a while after a baby is born. It should subside shortly." 4. "It is important for you to be examined by the doctor today. Let me check to see when you can come in." 4. "It is important for you to be examined by the doctor today. Let me check to see when you can come in." A client, 2 days postpartum from a spontaneous vaginal delivery, asks the nurse about postpartum exercises. Which of the following responses by the nurse is appropriate? 1. "You must wait to begin to perform exercises until after your six-week postpartum checkup." 2. "You may begin Kegel exercises today, but do not do any other exercises until the doctor tells you that it is safe." 3. "By next week you will be able to return to the exercise schedule you had during your prepregnancy." 4. "You can do some Kegel exercises today and then slowly increase your toning exercises over the next few weeks." Downloaded by Craig Cman ([email protected]) lOMoARcPSD|15450111 4. "You can do some Kegel exercises today and then slowly increase your toning exercises over the next few weeks." The nurse is examining a 2-day postpartum client whose fundus is 2 cm below the umbilicus and whose bright red lochia saturates about 4 inches of a pad in 1 hour. What should the nurse document in the nursing record? 1. Abnormal involution, lochia rubra heavy. 2. Abnormal involution, lochia serosa scant. 3. Normal involution, lochia rubra moderate. 4. Normal involution, lochia serosa heavy. 3. Normal involution, lochia rubra moderate. The nurse palpates a distended bladder on a woman who delivered vaginally 2 hours earlier. The woman refuses to go to the bathroom, "I really don't need to go." Which of the following responses by the nurse is appropriate? 1. "Okay. I must be palpating your uterus." 2. "I understand but I still would like you to try to urinate." 3. "You still must be numb from the local anesthesia." 4. "That is a problem. I will have to catheterize you." 2. "I understand but I still would like you to try to urinate." A client, G1P0101, postpartum 1 day, is assessed. The nurse notes that the client's lochia rubra is moderate and her fundus is boggy 2 cm above the umbilicus and deviated to the right. Which of the following actions should the nurse take first? 1. Notify the woman's primary health care provider. 2. Massage the woman's fundus. 3. Escort the woman to the bathroom to urinate. 4. Check the quantity of lochia on the peripad. 2. Massage the woman's fundus The nurse has taught a new admission to the postpartum unit about pericare.Which of the following indicates that the client understands the procedure? Downloaded by Craig Cman ([email protected]) lOMoARcPSD|15450111 1. The woman performs the procedure twice a day. 2. The woman sits in warm tap water for ten minutes. 3. The woman sprays her perineum from front to back. 4. The woman mixes tap water with hydrogen peroxide. 3. The woman sprays her perineum from front to back. The nurse informs a postpartum woman that ibuprofen (Advil) is especially effective for afterbirth pains. What is the scientific rationale for this? 1. Ibuprofen is taken every two hours. 2. Ibuprofen has an antiprostaglandin effect. 3. Ibuprofen is given via the parenteral route. 4. Ibuprofen is administered in high doses. 2. Ibuprofen has an antiprostaglandin effect. AD It is 4 p.m. A client, G1P0000, 3 cm dilated, asks the nurse when the dinner tray will be served. The nurse replies 1. "Laboring clients are never allowed to eat." 2. "Believe me, you will not want to eat by the time it is the dinner hour. Most women throw up, you know." 3. "The dinner tray should arrive in an hour or two." 4. "A heavy meal is discouraged. I can get clear fluids for you whenever you would like them, though." 4. "A heavy meal is discouraged. I can get clear fluids for you whenever you would like them, though." A physician has ordered an iron supplement for a postpartum woman. The nurse strongly suggests that the woman take the medicine with which of the following drinks? 1. Skim milk. 2. Ginger ale. 3. Orange juice. 4. Chamomile tea. Downloaded by Craig Cman ([email protected]) lOMoARcPSD|15450111 3. Orange juice. On admission to the labor and delivery unit, a client's hemoglobin (Hgb) was assessed at 11.0 gm/dL, and her hematocrit (Hct) at 33%. Which of the following values would the nurse expect to see 2 days after a normal spontaneous vaginal delivery? 1. Hgb 12.5 gm/dL; Hct 37%. 2. Hgb 11.0 gm/dL; Hct 33%. 3. Hgb 10.5 gm/dL; Hct 31%. 4. Hgb 9.0 gm/dL; Hct 27%. 3. Hgb 10.5 gm/dL; Hct 31%. During a postpartum assessment, it is noted that a G1P1001 woman, who delivered vaginally over an intact perineum, has a cluster of hemorrhoids. Which of the following would be appropriate for the nurse to include in the woman's health teaching? Select all that apply. 1. The client should use a sitz bath daily as a relief measure. 2. The client should digitally replace external hemorrhoids into her rectum. 3. The client should breastfeed frequently to stimulate oxytocin to reduce the size of the hemorrhoids. 4. The client should be advised that the hemorrhoids will increase in size and quantity with subsequent pregnancies. 5. The client should apply topical anesthetic as a relief measure. 1. The client should use a sitz bath daily as a relief measure. 2. The client should digitally replace external hemorrhoids into her rectum. 5. The client should apply topical anesthetic as a relief measure. Which of the following is the priority nursing action during the immediate postpartum period? 1. Palpate fundus. 2. Check pain level. Downloaded by Craig Cman ([email protected]) lOMoARcPSD|15450111 3. Perform pericare. 4. Assess breasts. 1. Palpate fundus. Immediately after delivery, a woman is shaking uncontrollably. Which of the following nursing actions is most appropriate? 1. Provide the woman with warm blankets. 2. Put the woman in Trendelenburg position. 3. Notify the primary health care provider. 4. Increase the intravenous infusion. 1. Provide the woman with warm blankets. One nursing diagnosis that a nurse has identified for a postpartum client is: Risk for intrauterine infection r/t vaginal delivery. During the postpartum period, which of the following goals should the nurse include in the care plan in relation to this diagnosis? Select all that apply. 1. The client will drink sufficient quantities of fluid. 2. The client will have a stable white blood cell count. 3. The client will have a normal temperature. 4. The client will have normal-smelling vaginal discharge. 5. The client will take two or three sitz baths each day. 2. The client will have a stable white blood cell count. 3. The client will have a normal temperature. 4. The client will have normal-smelling vaginal discharge. Which of the following nursing interventions would be appropriate for the nurse to perform in order to achieve the client care goal: The client will not develop postpartum thrombophlebitis? 1. Encourage early ambulation. 2. Promote oral fluid intake. 3. Massage the legs of the client twice daily. 4. Provide the client with high fiber foods. Downloaded by Craig Cman ([email protected]) lOMoARcPSD|15450111 1. Encourage early ambulation. The nurse is developing a plan of care for the postpartum client during the "taking in" phase. Which of the following should the nurse include in the plan? 1. Teach baby care skills like diapering. 2. Discuss the labor and birth with the mother. 3. Discuss contraceptive choices with the mother. 4. Teach breastfeeding skills like pumping. 2. Discuss the labor and birth with the mother. The nurse is developing a plan of care for the postpartum client during the "taking hold" phase. Which of the following should the nurse include in the plan? 1. Provide the client with a nutritious meal. 2. Encourage the client to take a nap. 3. Assist the client with activities of daily living. 4. Assure the client that she is an excellent mother. 4. Assure the client that she is an excellent mother. Which finding would the nurse view as normal when evaluating the laboratory reports of a 34-week gestation client? 1. Anemia. 2. Thrombocytopenia. 3. Polycythemia. 4. Hyperbilirubinemia. 1. Anemia. The nurse asks a 31-week gestation client to lie on the examining table during a prenatal examination. In which of the following positions should the client be placed? 1. Orthopneic. 2. Lateral-recumbent. Downloaded by Craig Cman ([email protected]) lOMoARcPSD|15450111 3. Sims'. 4. Semi-Fowler's. 4. Semi-Fowler's.. A third-trimester client is being seen for routine prenatal care. Which of the following assessments will the nurse perform during the visit? Select all that apply. 1. Blood glucose. 2. Blood pressure. 3. Fetal heart rate. 4. Urine protein. 5. Pelvic ultrasound. 2. Blood pressure. 3. Fetal heart rate. 4. Urine protein. A nurse is working in the prenatal clinic. Which of the following findings seen in third-trimester pregnant women would the nurse consider to be within normal limits? Select all that apply. 1. Leg cramps. 2. Varicose veins. 3. Hemorrhoids. 4. Fainting spells. 5. Lordosis. 1. Leg cramps. 2. Varicose veins. 3. Hemorrhoids. 5. Lordosis. A 36-week gestation gravid lies flat on her back. Which of the following maternal signs/symptoms would the nurse expect to observe? 1. Hypertension. 2. Dizziness. Downloaded by Craig Cman ([email protected]) lOMoARcPSD|15450111 3. Rales. 4. Chloasma. 2. Dizziness. The nurse is interviewing a 38-week gestation Muslim woman. Which of the following questions would be inappropriate for the nurse to ask? 1. "Do you plan to breastfeed your baby?" 2. "What do you plan to name the baby?" 3. "Which pediatrician do you plan to use?" 4. "How do you feel about having an episiotomy?" 2. "What do you plan to name the baby?" A woman is 36-weeks' gestation. Which of the following tests will be done during her prenatal visit? 1. Glucose challenge test. 2. Amniotic fluid volume assessment. 3. Vaginal and rectal cultures. 4. Karyotype analysis. 3. Vaginal and rectal cultures. A 34-week gestation woman calls the obstetric office stating, "Since last night I have had three nosebleeds." Which of the following responses by the nurse is appropriate? 1. "You should see the doctor to make sure you are not becoming severely anemic." 2. "Do you have a temperature?" 3. "One of the hormones of pregnancy makes the nasal passages prone to bleeds." 4. "Do you use any inhaled drugs?" 3. "One of the hormones of pregnancy makes the nasal passages prone to bleeds." AD Downloaded by Craig Cman ([email protected]) lOMoARcPSD|15450111 The nurse asks a woman about how the woman's husband is dealing with the pregnancy. The nurse concludes that counseling is needed when the woman makes which of the following statements? 1. "My husband is ready for the pregnancy to end so that we can have sex again." 2. "My husband has gained quite a bit of weight during this pregnancy." 3. "My husband seems more worried about our finances now than before the pregnancy." 4. "My husband plays his favorite music for my belly so the baby will learn to like it." 1. "My husband is ready for the pregnancy to end so that we can have sex again.". A man has inherited the gene for familial adenomatous polyposis (FAP), an autosomal dominant disease. He and his wife wish to have a baby. Which of the following would provide the couple with the highest probability of conceiving a healthy child? 1. Amniocentesis. 2. Chorionic villus sampling. 3. Pre-implantation genetic diagnosis. 4. Gamete intrafallopian transfer. 3. Pre-implantation genetic diagnosis. A woman asks the obstetrician's nurse about cord blood banking. Which of the following responses by the nurse would be best? 1. "I think it would be best to ask the doctor to tell you about that." 2. "The cord blood is frozen in case your baby develops a serious illness in the future." 3. "The doctors could transfuse anyone who gets into a bad accident with the blood." 4. "Cord blood banking is very expensive and the blood is rarely ever used." Downloaded by Craig Cman ([email protected]) lOMoARcPSD|15450111 2. "The cord blood is frozen in case your baby develops a serious illness in the future." A 3-month-old baby has been diagnosed with cystic fibrosis. The mother states, "How could this happen? I had an amniocentesis during my pregnancy and everything was supposed to be normal!" What does the nurse understand about this situation? 1. Cystic fibrosis cannot be diagnosed by amniocentesis. 2. The baby may have an uncommon genetic variant of the disease. 3. It is possible that the laboratory technician made an error. 4. Instead of obtaining fetal cells the doctor probably harvested maternal cells. 2. The baby may have an uncommon genetic variant of the disease. The nurse discusses the results of a 3-generation pedigree with the proband who has breast cancer. Which of the following information must the nurse consider? 1. The proband should have a complete genetic analysis done. 2. The proband is the first member of the family to be diagnosed. 3. The proband's first degree relatives should be included in the discussion. 4. The proband's sisters will likely develop breast cancer during their lives. 2. The proband is the first member of the family to be diagnosed. The nurse is analyzing the pedigree shown below. How should the nurse interpret the genotype of the individual in location II-4? 1. Affected male. 2. Unaffected female. 3. Stillborn child. 4. Child of unknown sex. 3. Stillborn child. The nurse is analyzing the pedigree shown below. How should the nurse interpret the genotype of the individuals in locations IV-9 and IV-10? 1. Fraternal twins. Downloaded by Craig Cman ([email protected]) lOMoARcPSD|15450111 2. Unaffected couple. 3. Proband and sister. 4. Known heterozygotes. 1. Fraternal twins. A woman who is a carrier for sickle cell anemia is advised that if her baby has two recessive genes, the penetrance of the disease is 100%, but the expressivity is variable. Which of the following explanations will clarify this communication for the mother? All babies with 2 recessive sickle cell genes will: 1. Develop painful vaso-occlusive crises during their first year of life. 2. Exhibit at least some signs of the disease while in the neonatal nursery. 3. Show some symptoms of the disease but the severity of the symptoms will be individual. 4. Be diagnosed with sickle cell trait but will be healthy and disease-free throughout their lives. 3. Show some symptoms of the disease but the severity of the symptoms will be individual. Analyze the pedigree below. Which of the following inheritance patterns does the pedigree depict? 1. Autosomal recessive. 2. Mitochondrial inheritance. 3. X-linked recessive. 4. Y-linked trait. 1. Autosomal recessive. A client, in her third trimester, is concerned that she will not know the difference between labor contractions and normal aches and pains of pregnancy. How should the nurse respond? 1. "Don't worry. You'll know the difference when the contractions start." 2. "The contractions may feel just like a backache, but they will come and go." Downloaded by Craig Cman ([email protected]) lOMoARcPSD|15450111 3. "Contractions are a lot worse than your pregnancy aches and pains." 4. "I understand. You don't want to come to the hospital before you are in labor." 2. "The contractions may feel just like a backache, but they will come and go." The nurse is counseling a pregnant couple who are both carriers for phenylketonuria (PKU), an autosomal recessive disease. Which of the following comments by the nurse is appropriate? 1. "I wish I could give you good news, but because this is your first pregnancy, your child will definitely have PKU." 2. "Congratulations, you must feel relieved that the odds of having a sick child are so small." 3. "There is a 2 out of 4 chance that your child will be a carrier like both of you." 4. "There is a 2 out of 4 chance that your child will have PKU." 3. "There is a 2 out of 4 chance that your child will be a carrier like both of you." To obtain the obstetric conjugate measurement, the nurse would do which of the following? 1. Add 1.5 cm to the transverse diameter. 2. First measure the angle of the pubic arch. 3. Subtract 1.5 to 2 cm from the diagonal conjugate. 4. Measure the diameter of the pelvic inlet. 3. Subtract 1.5 to 2 cm from the diagonal conjugate. The nurse is developing a teaching plan for a client entering the third trimester of her pregnancy. The nurse should include which of the following in the plan? Select all that apply. 1. Differentiating the fetus from the self. 2. Ambivalence concerning pregnancy. Downloaded by Craig Cman ([email protected]) lOMoARcPSD|15450111 3. Experimenting with mothering roles. 4. Realignment of roles and tasks. 5. Trying various caregiver roles. 6. Concern about labor and delivery. 3. Experimenting with mothering roles. 4. Realignment of roles and tasks. 5. Trying various caregiver roles. 6. Concern about labor and delivery. A new antenatal G 6, P 4, Ab 1 client attends her first prenatal visit with her husband. The nurse is assessing this couple's psychological response to their pregnancy. Which of the following requires the most immediate follow up? 1. The couple are concerned with financial changes this pregnancy causes. 2. The couple expresses ambivalence about the current pregnancy. 3. The father of the baby states that the pregnancy has changed the mother's focus. 4. The father of the baby is irritated that the mother is not like she was before pregnancy. 4. The father of the baby is irritated that the mother is not like she was before pregnancy. When preparing at endocrine changes that normally occur during pregnancy, the nurse should include information about which of the following subjects? 1. Human placental lactogen maintains the corpus luteum. 2. Progesterone is responsible for hyperpigmentation and vascular skin changes. 3. Estrogen relaxes smooth muscle in the respiratory tract. Downloaded by Craig Cman ([email protected]) lOMoARcPSD|15450111 4. The thyroid enlarges with an increase in basal metabolic rate. 4. The thyroid enlarges with an increase in basal metabolic rate. When developing a series of parent classes on fetal development, which of the following should the nurse include as being developed by the end of the third month (9 to 12 weeks)? 1. External genitalia. 2. Myelinization of nerves. 3. Brown fat stores. 4. Air ducts and alveoli. 1. External genitalia. A primigravid client attending parenthood classes tells the nurse that there is a history of twins in her family. What should the nurse tell the client? 1. Monozygotic twins result from fertilization of two ova by different sperm. 2. Monozygotic twins occur by chance regardless of race or heredity. 3. Dizygotic twins are usually of the same sex. 4. Dizygotic twins occur more often in primigravid than in multigravid clients. 2. Monozygotic twins occur by chance regardless of race or heredity. During a 2-hour childbirth preparation class focusing on the labor and delivery process for primigravid clients, the nurse is describing the maneuvers that the fetus goes through during the labor process when the head is the presenting part. In which order do these maneuvers occur? 1. Engagement 2. Descent Downloaded by Craig Cman ([email protected]) lOMoARcPSD|15450111 3. Flexion 4. Internal rotation 1,2,3,4 A primigravid client in a Preparation for Parenting class asks how much blood is lost during an uncomplicated delivery. The nurse should tell the woman: 1. "The maximum blood loss considered within normal limits is 500 mL." 2. "The minimum blood loss considered within normal limits is 1,000 mL." 3. "Blood loss during a delivery is rarely estimated unless there is a hemorrhage." 4. "It would be very unusual if you lost more than 100 mL of blood during the delivery." 1. "The maximum blood loss considered within normal limits is 500 mL." Which of the following statements by a primigravid client about the amniotic fluid and sac indicates the need for further teaching? 1. "The amniotic fluid helps to dilate the cervix once labor begins." 2. "Fetal nutrients are provided by the amniotic fluid." 3. "Amniotic fluid provides a cushion against impact of the maternal abdomen." 4. "The fetus is kept at a stable temperature by the amniotic fluid and sac." 2. "Fetal nutrients are provided by the amniotic fluid." During a childbirth preparation class, a primigravid client at 36 weeks' gestation tells the nurse, "My lower back has really been bothering me lately." Which of the following exercises suggested by the nurse would be most helpful? 1. Pelvic rocking. 2. Deep breathing. 3. Tailor sitting. 4. Squatting. 1. Pelvic rocking. Downloaded by Craig Cman ([email protected]) lOMoARcPSD|15450111 A client is experiencing pain during the first stage of labor. What should the nurse instruct the client to do to manage her pain? Select all that apply. 1. Walk in the hospital room. 2. Use slow chest breathing. 3. Request pain medication on a regular basis. 4. Lightly massage her abdomen. 5. Sip ice water. 1. Walk in the hospital room. 2. Use slow chest breathing. 4. Lightly massage her abdomen. During a Preparation for Parenting class, one of the participants asks the nurse, "How will I know if I am really in labor?" The nurse should tell the participant which of the following about true labor contractions? 1. "Walking around helps to decrease true contractions." 2. "True labor contractions may disappear with ambulation, rest, or sleep." 3. "The duration and frequency of true labor contractions remain the same." 4. "True labor contractions are felt first in the lower back, then the abdomen." 4. "True labor contractions are felt first in the lower back, then the abdomen." After instructing participants in a childbirth education class about methods for coping with discomforts in the first stage of labor, the nurse determines that one of the pregnant clients needs further instruction when she says that she has been practicing which of the following? 1. Biofeedback. 2. Effleurage. 3. Guided imagery. 4. Pelvic tilt exercises. 4. Pelvic tilt exercises. Downloaded by Craig Cman ([email protected]) lOMoARcPSD|15450111 After a Preparation for Parenting class session, a pregnant client tells the nurse that she has had some yellow-gray frothy vaginal discharge and local itching. The nurse's best action is to advise the client to do which of the following? 1. Use an over-the-counter cream for yeast infections. 2. Schedule an appointment at the clinic for an examination. 3. Administer a vinegar douche under low pressure. 4. Prepare for preterm labor and delivery. 2. Schedule an appointment at the clinic for an examination. The topic of physiologic changes that occur during pregnancy is to be included in a parenting class for primigravid clients who are in their first half of pregnancy. Which of the following would be important for the nurse to include in the teaching plan? 1. Decreased plasma volume. 2. Increased risk for urinary tract infections. 3. Increased peripheral vascular resistance. 4. Increased hemoglobin levels. 2. Increased risk for urinary tract infections The nurse is obtaining information to support the need for improved prenatal care services in the community. Which of the following information is most important to include? 1. The maternal mortality rate. 2. The infant mortality rate. 3. The perinatal mortality rate. 4. The neonatal mortality rate.. The maternal mortality rate. A multigravid client at 32 weeks' gestation has exprienced hemolytic disease of the newborn in a previous pregnancy. The nurse should prepare the client for frequent antibody titer evaluations obtained from which of the following? Downloaded by Craig Cman ([email protected]) lOMoARcPSD|15450111 1. Placental blood. 2. Amniotic fl uid. 3. Fetal blood. 4. Maternal blood.. Maternal blood. A client with a past medical history of ventricular septal defect repaired in infancy is seen at the prel clinic. She is complaining of dyspnea with exertion and being very tired. Her vital signs are 98, 80, 20, BP 116/72. She has + 2 pedal edema and clear breath sounds. As the nurse plans this client's care, which of the following is her cardiac classification according to the New York Heart Association Cardiac Disease classification? 1. Class I. 2. Class II. 3. Class III. 4. Class IV. 2. Class II. A primigravid client has completed her first prenatal visit and blood work. Her laboratory test for the hepatitis B surface antigen (HBsAg) is positive. The nurse can advise the client that the plan of care for this newborn will include? Select all that apply. 1. Hepatitis B immune globulin at birth. 2. Series of three hepatitis B vaccinations per recommended schedule. 3. Hepatitis B screening when born. 4. Isolation of infant during hospitalization. 5. Universal precautions for mother and infant. 6. Contraindication for breast-feeding because the mother is HBsAg positive. 1. Hepatitis B immune globulin at birth. 2. Series of three hepatitis B vaccinations per recommended schedule. 5. Universal precautions for mother and infant Downloaded by Craig Cman ([email protected]) lOMoARcPSD|15450111 A woman who has had asthma since she was a child and it is under control when the client takes her medication correctly and consistently is now pregnant for the first time. Which of the following client statements concerning asthma during pregnancy indicates the need for further instruction? 1. "I need to continue taking my asthma medication as prescribed." 2. "It is my goal to prevent or limit asthma attacks." 3. "During an asthma attack, oxygen needs continue to be high for mother and fetus." 4. "Bronchodilators should be used only when necessary because of the risk they present to the fetus." 4. "Bronchodilators should be used only when necessary because of the risk they present to the fetus." To decrease the possibility of a perineal laceration during delivery, the nurse performs which of the following interventions prior to the delivery? 1. Assists the woman into a squatting position. 2. Advises the woman to push only when she feels the urge. 3. Encourages the woman to push slowly and steadily. 4. Massages the perineum with mineral oil. 4. Massages the perineum with mineral oil. The nurse is performing a vaginal examination on a client in labor. The client is found to be 5 cm dilated, 90% effaced, and station 2. Which of the following has the nurse palpated? 1. Thin cervix. 2. Bulging fetal membranes. 3. Head at the pelvic outlet. 4. Closed cervix. 1. Thin cervix. A couple has delivered a 28-week fetal demise. Which of the following nursing actions are appropriate to take? Select all that apply. Downloaded by Craig Cman ([email protected]) lOMoARcPSD|15450111 1. Swaddle the baby in a baby blanket. 2. Discuss funeral options for the baby. 3. Encourage the couple to try to get pregnant again soon. 4. Ask the couple whether or not they would like to hold the baby. 5. Advise the couple that the baby's death was probably for the best. 1. Swaddle the baby in a baby blanket. 2. Discuss funeral options for the baby. 4. Ask the couple whether or not they would like to hold the baby. A client is being discharged on Coumadin (warfarin) post-pulmonary embolism after a cesarean delivery. Which of the following should be included in the patient teaching? 1. Only take ibuprofen for pain. 2. Avoid eating dark green leafy vegetables. 3. Drink grapefruit juice daily. 4. Report any decrease in urinary output. 2. Avoid eating dark green leafy vegetables. A client just delivered the placenta pictured below. For which of the following complications should the nurse carefully observe the woman? 1. Endometrial ischemia. 2. Postpartum hemorrhage. 3. Prolapsed uterus. 4. Vaginal hematoma. 2. Postpartum hemorrhage. Which of the following is a priority nursing diagnosis for a woman, G10P6226, who is PP1 from a spontaneous vaginal delivery with a significant postpartum hemorrhage? 1. Alteration is comfort related to afterbirth pains. 2. Risk for altered parenting related to grand multiparity. 3. Fluid volume deficit related to blood loss. 4. Risk for sleep deprivation related to mothering role. Downloaded by Craig Cman ([email protected]) lOMoARcPSD|15450111 3. Fluid volume deficit related to blood loss. A woman has just had a macrosomic baby after a 12-hour labor. For which of the following complications should the woman be carefully monitored? 1. Uterine atony. 2. Hypoprolactinemia. 3. Infection. 4. Mastitis. 1. Uterine atony. On admission to the labor and delivery suite, the nurse assesses the discharge needs of a primipara who will be discharged home 4 days after a cesarean delivery. Which of the following questions should the nurse ask the client? 1. "Have you ever had anesthesia before?" 2. "Do you have any allergies?" 3. "Do you scar easily?" 4. "Are there many stairs in your home?" 4. "Are there many stairs in your home?" A woman is receiving Paxil (paroxetine) for postpartum depression. In order to prevent a drug/food interaction, the client must be advised to refrain from consuming which of the following? 1. Alcohol. 2. Grapefruit. 3. Milk. 4. Cabbage. 1. Alcohol. A nurse is assessing a 1 day-postpartum client who had a spontaneous vaginal delivery over an intact perineum. The fundus is firm at the umbilicus, lochia moderate, and perineum edematous. One hour after receiving ibuprofen 600 mg po, the client is complaining of perineal pain at level 9 on a 10 point scale. Based on this information, which of the following is an Downloaded by Craig Cman ([email protected]) lOMoARcPSD|15450111 appropriate conclusion for the nurse to make about the client? 1. She should be assessed by her doctor. 2. She should have a sitz bath. 3. She may have a hidden laceration. 4. She needs a narcotic analgesic. 1. She should be assessed by her doctor. A breastfeeding mother calls the obstetrician's office with a complaint of pain in one breast. Upon inspection, a diagnosis of mastitis is made. Which of the following nursing interventions is appropriate? 1. Advise the woman to apply ice packs to her breasts. 2. Encourage the woman to breastfeed frequently. 3. Inform the woman that she should wean immediately. 4. Direct the woman to notify her pediatrician as soon as possible. 2. Encourage the woman to breastfeed frequently. A woman, who wishes to breastfeed, advises the nurse that she has had breast augmentation surgery. Which of the following responses by the nurse is appropriate? 1. Breast implants often contaminate the milk with toxins. 2. The glandular tissue of women who need implants is often deficient. 3. Babies often have difficulty latching to the nipples of women with breast implants. 4. Women who have implants are often able exclusively to breastfeed. 4. Women who have implants are often able exclusively to breastfeed. A breastfeeding client calls her obstetrician stating that her baby was diagnosed with thrush and that her breasts have become infected as well. Which of the following organisms has caused the baby's and mother's infection? 1. Staphylococcus aureus. 2. Streptococcus pneumonia. Downloaded by Craig Cman ([email protected]) lOMoARcPSD|15450111 3. Escherichia coli. 4. Candida albicans. 4. Candida albicans. A client is on magnesium sulfate via IV pump for severe preeclampsia. Other than patellar reflex assessments, which of the following noninvasive assessments should the nurse perform to monitor the client for early signs of magnesium sulfate toxicity? 1. Serial grip strengths. 2. Kernig assessments. 3. Pupillary responses. 4. Apical heart rate checks. 1. Serial grip strengths. A woman, 26 weeks' gestation, has just delivered a fetal demise. Which of the following nursing actions is appropriate at this time? 1. Remind the mother that she will be able to have another baby in the future. 2. Dress the baby in a tee shirt and swaddle the baby in a receiving blanket. 3. Ask the woman if she would like the doctor to prescribe a sedative for her. 4. Remove the baby from the delivery room as soon as possible. 2. Dress the baby in a tee shirt and swaddle the baby in a receiving blanket. A client, G1P0000, is PP1 from a normal spontaneous delivery of a baby boy, Apgar 5/6. Because the client exhibited addictive behaviors, a toxicology assessment was performed; the results were positive for alcohol and cocaine. Which of the following interventions is appropriate for this postpartum client? 1. Strongly advise the client to breastfeed her baby. 2. Perform hourly incentive spirometer respiratory assessments. 3. Suggest that the nursery nurse feed the baby in the nursery. 4. Provide the client with supervised instruction on baby care skills. 4. Provide the client with supervised instruction on baby care skills. Downloaded by Craig Cman ([email protected]) lOMoARcPSD|15450111 A client is 10 minutes postpartum from a forceps delivery of a 4500-gram Down syndrome neonate over a right mediolateral episiotomy. The client is at risk for each of the following nursing diagnoses. Which of the diagnoses is highest priority at this time? 1. Ineffective breastfeeding. 2. Fluid volume deficit. 3. Infection. 4. Pain. 2. Fluid volume deficit. A client is postpartum 24 hours from a spontaneous vaginal delivery with rupture of membranes for 42 hours. Which of the following signs/symptoms should the nurse report to the client's health care practitioner? 1. Foul-smelling lochia. 2. Engorged breasts. 3. Cracked nipples. 4. Cluster of hemorrhoids. 1. Foul-smelling lochia. A client is 36 hours post-cesarean section. Which of the following assessments would indicate that the client may have a paralytic ileus? 1. Abdominal striae. 2. Oliguria. 3. Omphalocele. 4. Absent bowel sounds. 4. Absent bowel sounds. A client, 1 day postpartum (PP), is being monitored carefully after a significant postpartum hemorrhage. Which of the following should the nurse report to the obstetrician? 1. Urine output 200 mL for last 8 hours. 2. Weight decrease of 2 pounds since delivery. Downloaded by Craig Cman ([email protected]) lOMoARcPSD|15450111 3. Drop in hematocrit of 2% since admission. 4. Pulse rate of 68 beats per minute. 1. Urine output 200 mL for last 8 hours. A nurse is working on the postpartum unit. Which of the following patients should the nurse assess first? 1. PP1 from vaginal delivery complains of burning on urination. 2. PP1 from forceps delivery with blood loss of 500 mL at time of delivery. 3. PP3 from vacuum delivery with hemoglobin of 7.2 g/dL. 4. PO3 from cesarean delivery complains of firm and painful breasts. 3. PP3 from vacuum delivery with hemoglobin of 7.2 g/dL. A nurse has administered Methergine (methylergonovine) 0.2 mg po to a grand multipara who delivered vaginally 30 minutes earlier. Which of the following outcomes indicates that the medication is effective? 1. Blood pressure 120/80. 2. Pulse rate 80 bpm and regular. 3. Fundus firm at umbilicus. 4. Increase in prothrombin time. 3. Fundus firm at umbilicus. A nurse on the postpartum unit is caring for two postoperative cesarean clients. One client had spinal anesthesia for the delivery while the other client had an epidural. Which of the following complications will the nurse monitor the spinal client for that the epidural client is much less high risk for? 1. Pruritus. 2. Nausea. 3. Postural headache. 4. Respiratory depression. 3. Postural headache. A postpartum woman has been diagnosed with postpartum psychosis. Which of the following signs/symptoms would the client exhibit? Downloaded by Craig Cman ([email protected]) lOMoARcPSD|15450111 1. Hallucinations. 2. Polyphagia. 3. Induced vomiting. 4. Weepy sadness. 1. Hallucinations. The nurse is providing discharge counseling to a woman who is breastfeeding her baby. What should the nurse advise the woman to do if she should palpate tender, hard nodules in her breasts? 1. Gently massage the areas toward the nipple especially during feedings. 2. Apply ice to the areas between feedings. 3. Bottlefeed for the next twenty-four hours. 4. Apply lanolin ointment to the areas after each and every breastfeeding. 1. Gently massage the areas toward the nipple especially during feedings. A woman states that all of a sudden her 4-day-old baby is having trouble feeding. On assessment, the nurse notes that the mother's breasts are firm, red, and warm to the touch. The nurse teaches the mother manually to express a small amount of breast milk from each breast. Which observation indicates that the nurse's intervention has been successful? 1. The mother's nipples are soft to the touch. 2. The baby swallows after every 5th suck. 3. The baby's pre- and postfeed weight change is 20 milliliters. 4. The mother squeezes her nipples during manual expression. 1. The mother's nipples are soft to the touch. A client's vital signs and reflexes were normal throughout pregnancy, labor, and delivery. Four hours after delivery the client's vitals are 98.6˚F, P 72, R 20, BP 150/100, and her reflexes are 4. She has an intravenous infusion running with 20 units of Pitocin (oxytocin) added. Which of the following actions by the nurse is appropriate? 1. Nothing because the results are normal. 2. Notify the obstetrician of the findings. Downloaded by Craig Cman ([email protected]) lOMoARcPSD|15450111 3. Discontinue the intravenous immediately. 4. Reassess the client after fifteen minutes. 2. Notify the obstetrician of the findings. A nurse is caring for a client, PP2, who is preparing to go home with her infant. The nurse notes that the client's blood type is O (negative), the baby's type is A (positive), and the direct Coombs' test is negative. Which of the following actions by the nurse is appropriate? 1. Advise the client to keep her physician appointment at the end of the week in order to receive her RhoGAM injection. 2. Carefully check the record to make sure that the RhoGAM injection was administered. 3. Notify the client that because her baby's Coombs' test was negative she will not receive an injection of RhoGAM. 4. Inform the client's physician that because the woman is being discharged on the second day, the RhoGAM could not be given. 2. Carefully check the record to make sure that the RhoGAM injection was administered. The nurse is caring for a couple who are in the labor/delivery room immediately after the delivery of a dead baby with visible defects. Which of the following actions by the nurse is appropriate? 1. Discourage the parents from naming the baby. 2. Advise the parents that the baby's defects would be too upsetting for them to see. 3. Transport the baby to the morgue as soon as possible. 4. Give the parents a lock of the baby's hair and a copy of the footprint sheet. 4. Give the parents a lock of the baby's hair and a copy of the footprint sheet. The nurse is circulating on a cesarean delivery of a G5P4004. All of the client's previous children were delivered via cesarean section. The physician Downloaded by Craig Cman ([email protected]) lOMoARcPSD|15450111 declares after delivering the placenta that it appears that the client has a placenta accreta. Which of the following maternal complications would be consistent with this diagnosis? 1. Blood loss of 2000 mL. 2. Blood pressure of 160/110. 3. Jaundice skin color. 4. Shortened prothrombin time. 1. Blood loss of 2000 mL. The nurse is providing care for a newborn who was delivered vaginally assisted by forceps. The nurse observes red marks on the head with swelling that does not cross the suture line. Which condition should the nurse documents in the medical record? Cephalhematoma A client at 34 weeks gestation comes to the birthing center complaining of vaginal bleeding that began one hour ago. The nurse assessment reveals approximately 30ML of bright red vaginal bleeding. Fetal rate of 130 - 140 beats per minute, no contractions and no complaints of pain what is the most likely cause of these client's bleeding. Placenta Previa A client at 30 weeks gestation reports that she has not felt the baby move in the last 24 hours. Concerned she arrives in a panic at the obstetric clinic where she is immediately sent to the hospital. which assessment warrants immediate intervention by the nurse. Fetal Heart rate 60 beats per minute AD Downloaded by Craig Cman ([email protected]) lOMoARcPSD|15450111 학습하지 않음 (47) 아직 이 단어들을 학습하지 않았어요! 47 개 선택 A client at 37 weeks gestation presents to labor and delivery with contractions every two minutes the nurse observes several shallow small vesicles on her pubis labia and perineum. the nurse should recognize the clients is prohibiting symptoms of which condition? Herpes Simplex Virus The nurse is planning care for a client at 30 weeks gestation who is experiencing preterm labor which maternity description is most important in preventing this fetus from developing respiratory distress syndrome. Betamethasone 12 mg deep IM A 16 year old gravida 1 para 0 client has just been admitted to the hospital with a diagnosis of eclampsia. She's not presently convulsing. Which intervention should the nurse plan to include in this client's nursing care plan? Keep an airway at the bedside At 12 hours after the birth of a healthy infant the mother complains of feeling constant vaginal pressure. The nurse determines the fundus is firm and at midline with moderate rubra lochia. which action should nurse take? Inspect clients perineal and rectal areas If primigravida at 36 weeks gestation who is RH negative experienced abdominal trauma in Downloaded by Craig Cman ([email protected]) lOMoARcPSD|15450111 a motor vehicle collision. Which assessment finding is most important for the nurse to report to the health care provider? Mild contractions every 10 minutes In The Ballard Gestational Age Assessment Tool, the nurse determines that a 15-month-old infant as a gestational age of 42 weeks. Based on this finding which intervention is most important for the nurse to implement. Provide a capillary blood glucose A new mother who is a lacto-ovo vegetarian plans to breast feed her infant. which information should the nurse provide prior to discharge. Continue prenatal vitamins with B12 While breastfeeding What should be the primary focus of nursing care in the transitional phase of Labor for a client who anticipates an unmedicated delivery. Assessing the strength of uterine contractions A care provider prescribes a maintenance dose of magnesium sulfate 2 grams per hour intravenously for clients with preeclampsia. The IV bag contains magnesium sulfate 20 grams how much in ml/Hr. should a nurse program the infusion pump enter numerical value only. 100 ml AD A client at 38 weeks gestation is admitted to labor and delivery with a complaint of contraction 5 minutes apart while the client is in the bathroom changing into a hospital gown the nurse hears the noise of a baby what should the nurse take first? Downloaded by Craig Cman ([email protected]) lOMoARcPSD|15450111 Push the call light for help The nurse is caring for a multiparous client who is 8 centimeters dilated 100% effaced and the fetal head is at 0 station. The clients is shivering and states extreme discomfort with the urge to bear down. which intervention should the nurse implement? Encourage pushing with each contraction. Following a traumatic delivery an infant receives an initial Apgar score of 3. which intervention is most important for the nurse to implement. Continue resuscitative efforts A 3-hour old male infants hands and feet as cyanotic, and has an axillary temperature of 96.5 degrees Fahrenheit 35.8 degrees centigrade a respiratory rate of 40 breaths per minute and a heart rate of 165 beats per minute what nursing action should nurse implement Gradually warm the infant under a radiant heat source. A new born nursery protocol includes a prescription for ophthalmic erythromycin 5% ointment to both eyes upon a new born admission. What action should the nurse take to ensure adequate installation of the client. Instill a thin ribbon into each lower conjunctival sac The nurse notes on the fetal monitor that a laboring client has a variable deceleration. which action should the nurse implement first. Change the client's position The nurse places one hand above the symphysis while massaging the fundus of a multiparous client who's uterine tone is boggy 15 minutes after delivering a Downloaded by Craig Cman ([email protected]) lOMoARcPSD|15450111 7 pounds 10 ounces 3220 grams infant which information should the nurse try to provide the client about those finding. Both the lower uterine segment and the fundus must be massaged A newborn assessment reveals spina bifida occulta. Which maternity factors should nurse identify as having the greatest impact on the development of this newborn complication. Folic acid deficiency A primigravida client in labor is receiving oxytocin 4 mu/minute to help promote an effective contraction pattern. The available solution is lactated ringer's 1,000 ml with oxytocin 20 units. The nurse should program the machine to deliver how many ML per hour. 12 mL AD A client who delivered a healthy newborn an hour ago asked the nurse when can she go home. Which information is most important for the nurse to provide the client. When there is no significant vaginal bleeding. A 17 year old client gave birth 12 hours ago she states that she doesn't know how to care for her baby. To promote parent infant attachment behaviors which intervention should the nurse implement. Encourage rooming in while in the hospital A pregnant client mentions in a history that she changes cats litter box daily. Which test should the nurse anticipate the health care provider to prescribe. Torch screening Downloaded by Craig Cman ([email protected]) lOMoARcPSD|15450111 The nurse is receiving report for a laboring client who arrived in the emergency center which ruptured membranes that the client did not recognize. Which is the priority nursing action to implement when the client his admitted to the labor and delivery suite? Monitor amniotic fluid for meconium. Four client at full term present to the labor and delivery unit at the same time. which client should a nurse access first. Multipara with contractions occurring every three minutes. The nurse is preparing to administer phytonadione to a newborn. Which statement makes made by the parents indicates understanding why the nurse is administering this medication. Prevent hemorrhagic disorders. The nurse is planning discharge teaching for four mothers. Which postpartum client is at highest risk for psychological difficulties during the postpartum period? A primiparous woman who has recently migrated to the US with a spouse. On the first postpartum day the nurse examines the breast of a new mother. Which condition is the nurse most likely to find. Filling and secreting colostrum. A client at 31 weeks gestation with a fundal height measurement of 25 c is scheduled for a series of ultrasounds to be performed every two weeks. Which explanation should the nurse provide. Evaluation of fetal growth. AD A primigravida client being treated for preeclampsia with magnesium sulfate delivered a 7 Downloaded by Craig Cman ([email protected]) lOMoARcPSD|15450111 pounds infant 4 hours ago by cesarean delivery. Which nursing problem has the highest priority? Risk for injury related to uterine atony Examination reveals that the laboring clients cervix is dilated to 2 centimeters, 70% effaced with the presenting part at -2 station the client tells the nurse I need my epidural now, this hurts, the nurses response to the client is based on which information. The client will need to be catheterized before the epidural can be administered. The mother of a breastfeeding 24-hour old infant is very concerned about the techniques involved in breastfeeding. She calls the nurse with each feeding to seek reassurance that she is doing it right she tells the nurse, "Now my daughter is not getting enough to eat" which response would be best for the nurse to make. If your baby's urine is straw colored, she's getting enough milk. A client in the first trimester of pregnancy calls the prenatal clinic to report she's nauseated, and her stools are black and thick since she started taking iron supplements last week. How should the nurse respond? select all that applies. Changes in color and consistency of stool are normal. A primiparous woman presents in labor with the following labs. hemoglobin 10.9 g/dl (109 g/dl) Hematocrit 29% (0.29) hepatitis surface antigen positive, Group B Streptococcus positive and rubella non-immune. which intervention should the nurse implement? Administer ampicillin 2 grams intravenously. Downloaded by Craig Cman ([email protected]) lOMoARcPSD|15450111 A mother spontaneously delivers a newborn infant in the taxicab while on the way to the hospital the emergency room nurse reported the mother as active herpes (H5V III) lesions on the vulva. Which intervention should the nurse implement first when admitting the neonate to the nursery? Place the newborn in the isolation area of the nursery. The health care provider prescribes 10 units per liters of oxytocin via IV drip to augment a client's labor because she's experiencing a prolonged active phase. Which finding would cause the nurse to immediately discontinue the oxytocin. Contraction duration of 100 seconds. A client who is 24 weeks gestation arrives to the clinic reporting swollen hands. On examination the nurse notes the clients as had a rapid weight gain over six weeks. which action should a nurse implements next? Obtain the clients blood pressure. the one minute Apgar score of a male infant, the nurse assesses a heart rate of 120 beats per minute and 41 respirations per minute. He has a loud cry with stimulation, good muscle tone and his color is ----------. What Apgar score should the nurse assign?... AD A multiparous client at 36 hours postpartum reports increased bleeding and cramping. On examination the nurse finds the uterine fundus 2 centimeters above the umbilicus. Which action should the nurse take first? Downloaded by Craig Cman ([email protected]) lOMoARcPSD|15450111 Call the HCP The nurse is scheduling a client with gestational diabetes for an amniocentesis because the fetus has an estimated weight of eight pounds 3629 grams at 36 weeks gestation. This amniocentesis is being performed to obtain which information Fetal lung maturity. A primigravida arrives at the observation unit of the maternity unit because she thinks she is in labor. The nurse applies the external fetal heart monitor and determines that the fetal heart rate is 140 beats per minute and contraction occurring irregularly every 10 to 15 minutes. Which assessment finding confirms to the nurse that the client is not in labor at this time. Contractions decrease with walking. A newborn assessment reveals spina bifida occulta. Which maternal factor should the nurse identify as having the greatest impact on the development of this newborn complication? Folic acid deficiency A 38-week primigravida is admitted to labor and delivery after a non-reactive result on a nonstress test (NST). The nurse begins a contraction stress test (CST) with an oxytocin infusion. Which finding is most important for the nurse to report to the health care provider. Spontaneous rupture of membranes. A newborn with a respiratory rate of 40 breaths per minute at one minute after birth is demonstrating cyanosis of the hands and feet. What action should a nurse take. Downloaded by Craig Cman ([email protected]) lOMoARcPSD|15450111 Continue to monitor client tells the nurse that she thinks she's pregnant. Which signs or symptoms provide the best indication that the client is pregnant. Hegar's sign. A newborns head circumference is 12 inches (30.5 cm) and his chest measurement is 13 inches (33 centimeters). The nurse notes that this infant has no molding, and it was a bridge presentation delivered by cesarean section. What action should the nurse take based on this data. Call these findings to the attention of the pediatrician. The head/chest ratio is abnormal. A 30-year-old primigravida delivers a nine-pound (4082 gram) infant vaginally after a 30-hour labor. What is priority nursing action for this client? Observe for signs of uterine hemorrhage A client with 26 weeks gestation was informed this morning that she has an elevated alpha fetal protein (AFP) level. After the health care provider leaves the room, the client asks what she should do next. What information should the nurse provide. Explain that his sonogram should be scheduled for definitive results. A woman who is 38 weeks gestation is receiving magnesium sulfate for severe preeclampsia. which assessment finding warrants immediate intervention by the nurse? Absent Patellar reflexes Downloaded by Craig Cman ([email protected]) lOMoARcPSD|15450111 Which finding for a client in labor at 41-weeks gestation requires additional assessment by the nurse? 1. Cervix dilated 2 cm and 50% effaced. 2. Score of 8 on the biophysical profile. 3. Fetal heart rate of 116 beats per minute. 4.One fetal movement noted in an hour. 4.One fetal movement noted in an hour. A client at 28-weeks gestation arrives at the labor and delivery unit with a complaint of bright red, painless vaginal bleeding. For which diagnostic procedure should the nurse prepare the client? 1. Contraction stress test. 2. Internal fetal monitoring. 3. Abdominal ultrasound. 4. Lecithin-sphingomyelin ratio. 3. Abdominal ultrasound 학습하지 않음 (52) 아직 이 단어들을 학습하지 않았어요! 52 개 선택 A multiparous client delivered a 7 lb 10 oz infant 5 hours ago. Upon fundal assessment, the nurse determines the uterus is boggy and is displaced above and to the right of the umbilicus. Which action should the nurse implement next? 1. Document the color of the lochia. 2. Observe maternal vital signs. 3. Assist the client to the bathroom. 4. Notify the healthcare provider 3. Assist the client to the bathroom. AD Downloaded by Craig Cman ([email protected]) lOMoARcPSD|15450111 A multiparous client is experiencing bleeding 2 hours after a vaginal delivery. What action should the nurse implement next? 1. Determine the firmness of the fundus 2. Give oxytocin (Pitocin) intravenously 3. Inform the healthcare provider of the bleeding. 4. Assess the vital signs for indicators of shock. 1. Determine the firmness of the fundus The nurse notes a pattern of the fetal heart rate decreasing after each contraction. What action should the nurse implement? 1. Give 10 liters of oxygen via face mask. 2. Prepare for an emergency cesarean section. 3. Continue to monitor the fetal heart rate pattern. 4. Obtain an oral maternal temperature 1. Give 10 liters of oxygen via face mask. A client at 28-weeks gestation experiences blunt abdominal trauma. Which parameter should the nurse assess first for signs of internal hemorrhage? 1. Vaginal bleeding. 2. Complaints of abdominal pain. 3. Changes in fetal heart rate patterns. 4. Alteration in maternal blood pressure. 3. Changes in fetal heart rate patterns. Which client should the nurse report to the healthcare provider as needing a prescription for Rh Immune Globulin (RhoGAM)? 1. Woman whose blood group is AB Rh-positive. 2. Newborn with rising serum bilirubin level. 3. Newborn whose Coombs test is negative. 4. Primigravida mother who is Rh-negative. 4. Primigravida mother who is Rh-negative. The nurse is caring for a client whose labor is being augmented with oxytocin (Pitocin). Which finding indicates that the nurse should discontinue the Downloaded by Craig Cman ([email protected]) lOMoARcPSD|15450111 oxytocin infusion? 1. The client needs to void. 2. Amniotic membranes rupture 3. Uterine contractions occur every 8 to 10 minutes. 4. The fetal heart rate is 180 bpm without variability 4. The fetal heart rate is 180 bpm without variability At 14-weeks gestation, a client arrives at the Emergency Center complaining of a dull pain in the right lower quadrant of her abdomen. The LPN/LVN obtains a blood sample and initiates an IV. Thirty minutes after admission, the client reports feeling a sharp abdominal pain and a shoulder pain. Assessment findings include diaphoresis, a heart rate of 120 beats/minute, and a blood pressure of 86/48. Which action should the nurse implement next? A. Check the hematocrit results. B. Administer pain medication. C. Increase the rate of IV fluids. D. Monitor client for contractions. C. Increase the rate of IV fluids. During a prenatal visit, the LPN/LVN discusses with a client the effects of smoking on the fetus. When compared with nonsmokers, mothers who smoke during pregnancy tend to produce infants who have A. lower Apgar scores. B. lower birth weights. C. respiratory distress. D. a higher rate of congenital anomalies. D. a higher rate of congenital anomalies. Which action should the LPN/LVN implement when preparing to measure the fundal height of a pregnant client? A. Have the client empty her bladder. B. Request the client lie on her left side. Downloaded by Craig Cman ([email protected]) lOMoARcPSD|15450111 C. Perform Leopold's maneuvers first. D. Give the client some cold juice to drink. A. Have the client empty her bladder. The LPN/LVN identifies crepitus when examining the chest of a newborn who was delivered vaginally. Which further assessment should the nurse perform? A. Elicit a positive scarf sign on the affected side. B. Observe for an asymmetrical Moro (startle) reflex. C. Watch for swelling of fingers on the affected side. D. Note paralysis of affected extremity and muscles. B. Observe for an asymmetrical Moro (startle) reflex. AD One hour after giving birth to an 8-pound infant, a client's lochia rubra has increased from small to large and her fundus is boggy despite massage. The client's pulse is 84 beats/minute and blood pressure is 156/96. The healthcare provider prescribes Methergine 0.2 mg IM Å~ 1. What action should the LPN/LVN take immediately? A. Give the medication as prescribed and monitor for efficacy. B. Encourage the client to breastfeed rather than bottle feed. C. Have the client empty her bladder and massage the fundus. D. Call the healthcare provider to question the prescription. D. Call the healthcare provider to question the prescription. Methergine is contraindicated for clients with elevated blood pressure, so the nurse should contact the healthcare provider and question the prescription (D). The LPN/LVN is preparing to give an enema to a laboring client. Which client requires the most caution when carrying out this procedure? A. A gravida 6, para 5 who is 38 years of age and in early labor. B. A 37-week primigravida who presents at 100% effacement, 3 cm cervical dilatation, and a -1 station. C. A gravida 2, para 1 who is at 1 cm cervical dilatation and a 0 station Downloaded by Craig Cman ([email protected]) lOMoARcPSD|15450111 admitted for induction of labor due to post dates. D. A 40-week primigravida who is at 6 cm cervical dilatation and the presenting part is not engaged. D. A 40-week primigravida who is at 6 cm cervical dilatation and the presenting part is not engaged. When the presenting part is ballottable (D), it is floating out of the pelvis. In such a situation, the cord can descend before the fetus causing a prolapsed cord, which is an emergency situation. A client at 32-weeks gestation comes to the prenatal clinic with complaints of pedal edema, dyspnea, fatigue, and a moist cough. Which question is most important for the LPN/LVN to ask this client? A. Which symptom did you experience first? B. Are you eating large amounts of salty foods? C. Have you visited a foreign country recently? D. Do you have a history of rheumatic fever? D. Do you have a history of rheumatic fever? Clients with a history of rheumatic fever (D) may develop mitral valve prolapse, which increases the risk for cardiac decompensation due to the increased blood volume that occurs during pregnancy, so obtaining information about this client's health history is a priority. The LPN/LVN is assessing a client who is having a non-stress test (NST) at 41weeks gestation. The nurse determines that the client is not having contractions, the fetal heart rate (FHR) baseline is 144 bpm, and no FHR accelerations are occurring. What action should the nurse take? A. Check the client for urinary bladder distention. B. Notify the healthcare provider of the nonreactive results. C. Have the mother stimulate the fetus to move. D. Ask the client if she has felt any fetal movement. D. Ask the client if she has felt any fetal movement. Downloaded by Craig Cman ([email protected]) lOMoARcPSD|15450111 The LPN/LVN identifies crepitus when examining the chest of a newborn who was delivered vaginally. Which further assessment should the nurse perform? A. Elicit a positive scarf sign on the affected side. B. Observe for an asymmetrical Moro (startle) reflex. C. Watch for swelling of fingers on the affected side. D. Note paralysis of affected extremity and muscles. B. Observe for an asymmetrical Moro (startle) reflex. A mother brings her 8 mo. old baby boy to clinic bc he has been vomitting and had diarrhea for last 3 days. Which assessment is most important for nurse to make? a. Assess infant abdomen for tenderness b. Determine if the infant was exposed to a virus c. Measure the infant's pulse d. Evaluate the infant's cry c. Measure the infant's pulse While obtaining the vital signs of a 10 year old who had a tonsillectomy this morning, the nurse observes the child swallowing every 2-3 minutes. Which assessment should the nurse implement? a. Inspect the posterior oropharynx b. Assess for teeth clenching or grinding c. Touch the tonsillar pillars to stimulate the gag reflex d. Ask the child to speak to evaluate change in voice tone a. Inspect the posterior oropharynx The parents of a 3-year old boy who has Duchenne muscular dystrophy ask, "How can our son have this disease? We are wondering if we should have any more children." What information should the nurse provide to parents? a. This is an inherited X-linked recessive disorder, which primarily affects male children in the family b. The striated muscle groups of males can be impacted by a lack of the protein dystrophin in their mothers Downloaded by Craig Cman ([email protected]) lOMoARcPSD|15450111 c. The male infant had a viral infection that went unnoticed and untreated so muscle damage was incurred d. Birth trauma with a breech vaginal birth causes damage to the spinal cord, thus weakening the muscles a. This is an inherited X-linked recessive disorder, which primarily affects male children in the family A 2-week-old female infant is hospitalized for the surgical repair of an umbilical hernia. After returning to the postoperative neonatal unit, her RR and HR have increased during the last hour. Which intervention should the nurse implement? a. Notify the HCP of these findings b. Administer a PRN analgesic prescription c. Record the findings in the child's record d. Wrap the infant tightly and rock in rocking chair b. Administer a PRN analgesic prescription AD A 2-year-old girl is brought to the clinic by her 17 year old mother. When the nurse observes that the child is drinking sweetened soda from her bottle, what information should the nurse discuss with this mother? Select all that apply A 2-year old should be speaking in 2 word phrases b. Dental caries are associated with drinking soda c. Drinking soda is related to childhood obesity d. Toddlers should be sleeping 10 hours a night e. Toddlers should be drinking from a cup by age 2 b. Dental caries are associated with drinking soda e. Toddlers should be drinking from a cup by age 2 A mother brings her 3 month old infant to the clinic because the baby does not sleep through the night. Which finding is most significant in planning care for this family? a. The mother is a single parent and lives with her parents Downloaded by Craig Cman ([email protected]) lOMoARcPSD|15450111 b. The mother states the baby is irritable during feedings c. The infant's formula has been changed twice d. The diaper area shows severe skin breakdown d. The diaper area shows severe skin breakdown The nurse determines that an infant admitted for surgical repair of an inguinal hernia voids a urinary stream from the ventral surface of the penis. What action should the nurse take? a. Document the finding b. Palpate scrotum for testicular descent c. Assess for bladder distension d. Auscultate bowel sounds a. Document the finding A 16 year old with acute myelocytic leukemia is receiving chemotherapy (CT) via an implanted medication port at the out-patient oncology clinic. What action should the nurse implement when the infusion is complete? a. Administer Zofran b. Obtain blood samples for RBCs, WBCs, and platelets c. Flush mediport w/ saline and heparin solution d. Initiate an infusion of normal saline c. Flush mediport w/ saline and heparin solution A mother brings her 3-week old infant to the clinic because the baby vomits after eating and always seems hungry. Further assessment indic

Use Quizgecko on...
Browser
Browser